16
$\begingroup$

Let $p > 3$ be a prime number, and $\zeta$ be a primitive $p$-th root of unity. I am interested in knowing the exact value of $$w_p = \prod_{a \in (\mathbb F_p^{\times})^2}(1 + \zeta^a) + \prod_{b \in \mathbb F_p^{\times} \backslash (\mathbb F_p^{\times})^2}(1 + \zeta^b). $$ (The term $w_p$ is related to the number of subsets $S \subset \mathbb F_p$ for which $\sum_{s \in S} s^2 = 0.$)

It is more or less straightforward to prove that $w_p = 2$ for $p \equiv \pm 1 \ (\text{mod}. 8)$ and $w_p = -2$ for $p \equiv 3 \ (\text{mod}. 8).$

(If anyone wants to see a proof for that, I can provide it. It boils down to the fact that $2 \in (\mathbb F_p^{\times})^2$ in the first case and $-1 \not\in (\mathbb F_p^{\times})^2$ in the scond case.)

So it remains to evealuate $w_p$ for $p \equiv 5 \ (\text{mod}. 8)$. One might conjecture that in this case, we also have $w_p = -2$, but this is wrong: The values for $p = 5,13,29,37$ are $w_p = 3,11,27,146$.

Indeed, it is easy to show that also in this case $w_p$ is an integer and $w_p \equiv -2 \ (\text{mod}. p)$. Looking at the values for small $p$, I arrived at the following conjecture:

Suppose $p \equiv 5 \ (\text{mod}. 8)$. Let $(\alpha,\beta)$ be the fundamental positive solution of the generalized Pell equation $\alpha^2 - p \beta^2 = 4$. Then $w_p = \alpha$.

Can anyone tell me if this statement is true?

$\endgroup$

1 Answer 1

27
$\begingroup$

According to Theorem 5.1 of http://math.mit.edu/~rstan/pubs/pubfiles/35.pdf, the number is $$ \frac 1p\left[ 2^{p-1}+\frac{p-1}{2}(\epsilon^{4h} + \epsilon^{-4h})\right], $$ where $h$ is the class number of $\mathbb{Q}(\sqrt{p})$ and $\epsilon>1$ the fundamental unit of $\mathbb{Q}(\sqrt{p})$.

Addendum. I should clarify that the number above is the number of subsets $S$ of $\mathbb{F}_p^*$ satisfying $\sum_{s\in S}s^2=0$. To get the number of such subsets of $\mathbb{F}_p$, multiply by two.

$\endgroup$
1
  • $\begingroup$ This is great; it gives (if I did not do a mistake) that $w_p = \epsilon^{2h} + \epsilon^{-2h}$. So it seems that my conjectrue is wrong, but there is a nice closed formula for $w_p$. $\endgroup$ Apr 19, 2015 at 21:39

Your Answer

By clicking “Post Your Answer”, you agree to our terms of service and acknowledge you have read our privacy policy.

Not the answer you're looking for? Browse other questions tagged or ask your own question.